1
$\begingroup$

This question can be seen as a variant of the post Bounded density for diffusions with diffusion coefficients bounded away from $0$ by Iosif Pinelis. Namely, consider the diffusion

$$X_t=\int_0^t a(s,X_s){\bf 1}_{\{|X_s|<1\}}dW_s,\quad \forall t\ge 0,$$

where $(W_t)_{t\ge 0}$ is a standard Brownian motion and $a$ is smooth s.t. $\inf_{(t,x)}a(t,x)\ge c>0$. Denote by $\mu_t$ the distribution of $X_t$. Can we write (under suitable conditions)

$$\mu_t(dx) = q^+_t\delta_{1}(dx)+q^-_t\delta_{-1}(dx)+ p_t(x)dx?$$

Here $q^{\pm}_t=\mathbb P[X_t=\pm 1]$ and $p_t$ (up to a normalization) is the conditional density function of $X_t$ knowing $\{\tau>t\}$, where $\tau:=\inf\{t\ge 0: |X_t|\ge 1\}$. So an alternative formulation is whether $X_t$ admits a density on the event $\{\tau>t\}$.

Any solution, references or comments are appreciated.

$\endgroup$

1 Answer 1

1
$\begingroup$

The answer is yes. Indeed, let $$Y_t=\int_0^t a(s,Y_s)\,dW_s\quad \forall t\ge 0.$$ Then $X_t=Y_t$ on the event $\{\tau>t\}$. So, for any Borel set $A\subseteq(-1,1)$, we have $$P(X_t\in A)=P(Y_t\in A,\tau>t)\le P(Y_t\in A).$$ So, the distribution of $X_t$ is absolutely continuous with respect to the distribution of $Y_t$. By the previous answer, for $t>0$, the distribution of $Y_t$ has a density (with respect to the Lebesgue measure). Thus, for $t>0$, the distribution $\mu_t$ of $X_t$ has a density $p_t$ on the interval $(-1,1)$ as well, so that indeed $$\mu_t(dx) = q^+_t\delta_{1}(dx)+q^-_t\delta_{-1}(dx)+ p_t(x)dx$$ for some nonnegative $q^\pm_t$.

$\endgroup$
3
  • $\begingroup$ Very tricky inequality! Thanks a lot Iosif for your answer $\endgroup$
    – GJC20
    Nov 1, 2021 at 16:51
  • $\begingroup$ Btw, is it known that this density $p_t$ is related to any PDE (as shown in the paper that you found previously)? $\endgroup$
    – GJC20
    Nov 1, 2021 at 16:58
  • $\begingroup$ @GJC20 : I vaguely remember seeing some work on this long ago, but this is all I remember in this regard, unfortunately. $\endgroup$ Nov 1, 2021 at 17:01

Your Answer

By clicking “Post Your Answer”, you agree to our terms of service and acknowledge you have read our privacy policy.

Not the answer you're looking for? Browse other questions tagged or ask your own question.